I need who help .. who can be my lifesaver

I Need Who Help .. Who Can Be My Lifesaver

Answers

Answer 1

Answer:

Q = G

Step-by-step explanation:

We are already given that angle P = angle H

We are also given that side QP = side GH

Remember if two sides are congruent then so are their opposite angles meaning that the opposite angle of GH ( which would be angle F ) would be congruent to the opposite angle of QP ( which would be angle R )

The remaining angles would be angle q and angle g so the additional information needed would be G = Q


Related Questions

She walks to her school at a distance of 800m from her house in 55 seconds. On reaching, she finds that the school is closed and comes back by car with her friend and reaches home in 25 seconds. Find her average speed in m/s

Answers

Answer:

per second she was walking 14.54m roughly and driving is 32m per second therefore the average is 23.27m/s

Step-by-step explanation:

divide both sides by 55 to get one.

divide both sides by 25 to get one

THEN

to find the average you simply add them and divide by two which is 46.54 which divided by two is roughly 23.27.

Samira and Sonia each have a bag containing 20 sweets. In each bag, there are 5 red, 6 green and 9 yellow sweets.
(a)
(b)
Samira chooses one sweet at random from her bag.
Write down the probability that she chooses a yellow sweet.
[1] Sonia chooses two sweets at random, without replacement, from her bag.
(i) Show that the probability that she chooses two green sweets is 3 . 38
(ii) Calculate the probability that the sweets she chooses are not both the same colour.

Answers

Step-by-step explanation:

there must be some typos here. but I try to address the right issues.

there is nothing under (a). so, really nothing to answer.

(b)

there are 20 sweets in her bag. 9 if the 20 are yellow.

so, the probability is the ratio of desired possibilities vs. total possibilities.

that is, tada! 9/20

that is the probabilty for Samira to pick a yellow sweet.

[1] (i)

again, 20 sweets to start with.

6 are green.

when she picks the first one, her probability to pick a green one is 6/20 = 3/10 = 0.3

and now, under the assumption that this came true, she picks another sweet.

this time she had only 19 left, and 5 of them are green.

so, this probabilty is 5/19

now both events need to happen for the case we are discussing. there is no overlapping, no ors, ifs and buts. it is just the product of both probabilities.

3/10 × 5/19 = 15/190 = 3/38

I think that is what the description asks for.

(ii)

that probability is

first selection is red and second is not red +

first selection is green and second is not green +

first selection is yellow and second is not yellow

so,

red and not red

5/20 = 1/4 red

15/19 not red (there are still 15 sweets of other colors in the bag, but again now only 19 total).

red and not red = 1/4 × 15/19 = 15/76 = 0.1974

green and not green

6/20 = 3/10 = green

14/19 = not green

green and not green = 3/10 × 14/19 = 52/190 = 26/95 =

= 0.2737

yellow and not yellow

9/20 = yellow

11/19 = not yellow

yellow and not yellow = 9/20 × 11/19 = 99/380 = 0.2606

so, now assuming up all 3 possibilities ("or" = sum) gives us the general possibility of selecting two different colors

= 0.7316

7
Setting 3 equal to which ratio would result in a valid proportion?
9
49
18
42
织但

Answers

Answer:

000

Step-by-step explanation:

000

Pls help!! find the area of the shaded region.

Answers

Answer:

134.1

Step-by-step explanation:

Area of the circle = 49π = 153.9 (rounded to the nearest tenth)

Segment area,

49/2(150π/360-sin(150))

= 19.8 (rounded to the nearest tenth)

Subtracting them,

153.9-19.8

= 134.1 cm²

Answered by GAUTHMATH

The area of the shaded region is 134.1 cm²

What is a segment of a circle?

'A segment of a circle is the region that is bounded by an arc and a chord of the circle.'

According to the given problem,

Area of the circle = πr²

                              = π × 7 × 7 cm²

                              = 153.9 (rounded to the nearest tenth)

Area of the Shaded region,

= [tex]\frac{r^{2} }{2}( \frac{angle in degrees * \pi }{360 - sin(angle in degrees)} )[/tex]

=[tex]\frac{49}{2}(\frac{150\pi }{360 - sin(150)})[/tex]

= 19.8 (rounded to the nearest tenth)

Subtracting them,

= 153.9 - 19.8

= 134.1 cm²

Hence, we can conclude that the area of the shaded region is 134.1cm²

Learn more about segment of a circle here: https://brainly.com/question/4910703

#SPJ2

The Yogurt Packaging company is now re-designing their factory. They are going to construct a building in
the shape of a square box. Their contractor has told them that the diagonal braces needed to make the roof
are 150 feet long.(please help)


1. What would be the floor plan of the building? Label the diagonal braces



2. Find the dimensions of the new factory



3.Find the perimeter of the new factory


4.the yogurt machines will be placed along the wall of the new factory . Each machine is 10 feet wide.

Find the company fit 40 of the machines in the building Why or Why not ?

Answers

Answer:

1.  see attached diagram

2. The dimensions are 75 sqrt(2) by 75 sqrt(2) ft

3. The perimeter is 300 sqrt(2) ft

4. cannot fit 40 machines along the walls

Step-by-step explanation:

We are using a top down view.  We have a square building with side length s.  The diagonal is 150 ft

Using the Pythagorean theorem

s^2 + s^2 = 150^2

2s^2 = 22500

s^2 =11250

Taking the square root of each side

sqrt(s^2) = sqrt(11250)

s = sqrt(6225*2)

s = 75 sqrt(2)

The dimensions are 75 sqrt(2) by 75 sqrt(2) ft

The perimeter of a square is given by

P = 4s = 4(75 sqrt(2)) =300 sqrt(2) ft

The perimeter is 300 sqrt(2) ft

Assuming the machines are square ( that they are as wide as they are long) 75 sqrt(2) is approximately 106 ft  so you can fit 10 along the wall

Putting them along the top and bottom walls = 20 machines

We are only left with 86 ft along the side walls

86/10 = 8 machines  

machines * 2 walls = 16

We can fit 20+16 machines = 36 machines not 40

In the adjoining fig. In a circle with centre C and chord DE ,ahe CF perpendicular to chord DE.If diameter of a circle is 20cm and DE=16cm,then CF=?GIVE REASON​

Answers

DC=CF

Diameter=20cmRadius=20/2=10cm

We know

[tex]\boxed{\sf L=\dfrac{\theta}{180}\times πr}[/tex]

[tex]\\ \large\sf\longmapsto L=\dfrac{90}{180}\times \dfrac{22}{7}\times 10[/tex]

[tex]\\ \large\sf\longmapsto L=\dfrac{1}{2}\times \dfrac{220}{7}[/tex]

[tex]\\ \large\sf\longmapsto L=\dfrac{110}{7}[/tex]

[tex]\\ \large\sf\longmapsto L=15.5[/tex]

Now

[tex]\\ \large\sf\longmapsto CF=L+r-DE[/tex]

[tex]\\ \large\sf\longmapsto CF=15.7+10-16[/tex]

[tex]\\ \large\sf\longmapsto CF=25.7-16[/tex]

[tex]\\ \large\sf\longmapsto CF=9.7cm[/tex]

[tex]\large\sf\red{⟼L=18090×722×10}[/tex]

[tex]\begin{gathered}\\ \large\sf\red{ ⟼ L=\dfrac{1}{2}\times \dfrac{220}{7}}\end{gathered}[/tex]

[tex]\begin{gathered}{ \large\sf\red{⟼ L=\dfrac{110}{7}}\end{gathered}[/tex]

L=15.5

Now

CF=L+r−DE

CF=15.7+10−16

CF=25.7−16

⟼CF=9.7cm

n-9 = 13
------------------

Answers

Answer:

n-9=13

n=22

Step-by-step explanation:

Answer: n = 22

Step-by-step explanation: to get the answer you have to add 9 to the -9 so that it cancels out. then you are to add the 9 to the 13 and so the answer shall be n=22

A tax on which of these products or services would not be considered a sin
tax"?
O A. Alcohol
O B. Gambling
O C. Electronics
O D. Tobacco

Answers

Answer: electronics

Hope that helps

The option that is not considered a sin tax is option B, gambling.

What is a sin tax?

A sin tax is a tax that is applied to "harmful things" to society or individuals. An example of this is Tobacco.

From the given options, the harmful ones are alcohol, gambling, and tobacco. These 3 are considered harmful, so would be included under the sin taxes.

Then the option that is not considered a sin tax is option B, gambling.

If you want to learn more about taxes:

https://brainly.com/question/25783927

#SPJ5

Which equation represents a line which is parallel to the line 6x - 5y = 5?
Submit Answer
Oy= $– 2
Oy= {2+3
Oy=+*+1
O 9= -x +6
PLS HELP ASAP

Answers

Answer:

y = 6/5x - 2

Step-by-step explanation:

First we need to put the equation given in slope intercept form, y = mx + b

6x - 5y = 5

Solve for y

6x - 5y = 5

Subtract 6x from both sides

-5y = -6x + 5

Divide both sides by -5

y = 6/5x - 1

Now we have the equation in slope intercept form

y = 6/5x - 1

We want to find the equation that would be parallel to it

Parallel lines have the same slope

The equation y = 6/5x - 1 has a slope of 6/5 so an equation that is parallel to it would also have a slope of 6/5

The only equation that has a slope of 6/5 is the top left one therefore that is the answer

Answer:

y = 6/5x - 2

Step-by-step explanation:

Simplify the given equation:

[tex]6x-5y=5\\-5y=-6x+5\\y=\frac{6}{5}x-1[/tex]

In order for another equation to be parallel, it must have the same slope. Therefore, y = 6/5x - 2 is the correct answer.

For the Parabola y = (x - 7)^2 – 3. the equation for the Line Of Symmetry is

Answers

Answer:

x+=(y)=x

Step-by-step explanation:

Answer:

x = 7

Step-by-step explanation:

y = (x - 7)^2 – 3

This equation is in vertex form

y = a(x-h)^2 +k  where (h,k) is the vertex.  For a vertical parabola, the axis of symmetry is x=h

x = 7

Write an equation in point-slope form for the line through the given point with the given slope. (Need ASAP help)
Is it a, b, c or d?

Answers

A because le puede hacer el pedido para la que oop no se w no

Can someone help me solve please
[tex]x + 2 \sqrt{x} - 3 = 0[/tex]

Answers

Answer:

x = 1

Step-by-step explanation:

Given

x + 2[tex]\sqrt{x}[/tex] - 3 = 0 ( subtract x - 3 from both sides )

2[tex]\sqrt{x}[/tex] = 3 - x ( square both sides )

4x = (3 - x)² ← expand using FOIL

4x = 9 - 6x + x² ( subtract 4x from both sides )

0 = x² - 10x + 9 ← in standard form

0 = (x - 1)(x - 9) ← in factored form

Equate each factor to zero and solve for x

x - 1 = 0 ⇒ x = 1

x - 9 = 0 ⇒ x = 9

As a check

Substitute these values into the left side of the equation and if equal to the right side then they are solutions.

x = 1 : 1 + 2[tex]\sqrt{1}[/tex] - 3 = 1 + 2 - 3 = 0 → x = 1 is a solution

x = 9 : 9 + 2[tex]\sqrt{9}[/tex] - 3 = 9 + 6 - 3 = 12 ≠ 0

x = 9 is an extraneous solution while x = 1 is a solution

PLS HELP ME ON THIS QUESTION I WILL MARK YOU AS BRAINLIEST IF YOU KNOW THE ANSWER!!
Find the interquartile range of the data set that consists of 3, 11, 4, 3, 10, 6, 4, 5.

A. 3.5
B. 5.75
C. 4.5
D. 8

Answers

Answer:

I am thinking it's c

I am not sure

Answer:

25th Percentile: 3.5

50th Percentile: 4.5

75th Percentile: 8

= Interquartile Range: 4.5

I think it shows it's '4.5'.

C

Determine the slope of the line that has the following coordinates: (-2, 5) (4, -2)

Answers

Answer:

slope= -1  1/6

Step-by-step explanation:

x1 y1  x2 y2

-2 5  4 -2

   

(Y2-Y1) (-2)-(5)=   -7  ΔY -7

(X2-X1) (4)-(-2)=    6  ΔX 6

   

slope= -1  1/6    

B= 2  2/3    

What is the smallest prime number that is also a multiple of 20?

Answers

Answer:

Hello,

Step-by-step explanation:

In order to cut off ( pour couper court)

all multiple of 20 are divisible by 20, so they are not primes.

A spinner has 4 equally likely outcomes: A,B,C, and D. The spinner is spun twice. What is the probability that it lands on BB twice?

Answers

Answer:

6.25%

Step-by-step explanation:

The probability of landing on B once is 25%.

So you multiply 25% by 25%.

0.25×0.25

The answer is 0.0625.

Converting back to %, its 6.25%.

I hope this helps!

pls ❤ and mark brainliest pls!

Tony gets married next month. One year ago from the date he will get married, Tony was away in
Spain for New Year's Eve. What month is it?

Answers

Answer:

late January or early February

Step-by-step explanation:

On using logical reasoning, we deduce that this month is November.

What is logical reasoning?

Logical reasoning comprises aptitude problems that need a logical level of examination to arrive at the right answer.

One year ago from the date he will get married, Tony was away in Spain for New Year's Eve.

This implies, One year ago from the date he will get married, Tony was away in Spain in month of December.

This implies, Tony will get married in month of December.

We know, Tony gets married next month. So, this month is November.

Learn more about logical reasoning here

https://brainly.com/question/2848937

#SPJ2

round to the nearest underlined place 18 904 A. 18,000 B. 19,000 C. 20,000 D. 18,920

Answers

Answer:

B 19,000

Step-by-step explanation:

;------------------------;

For people who have taken PreCalc/Calc (Summer Math Packet)
Find the constant a such that the function is continuous on the entire real line:

Answers

There is no a that will make f(x) continuous.

No matter what value of a you pick, you will always have the one-sided limits,

[tex]\displaystyle \lim_{x\to a^-}f(x) = \lim_{x\to a}(x^2-a^2) = 0[/tex]

[tex]\displaystyle \lim_{x\to a^+}f(x) = \lim_{x\to a}(x^2-a^2) = 0[/tex]

but f(a) = 8 regardless of a.

Calculation: One score is 20. Two score is 40. Three score and 5 is 65.
How many is four score and 7?

Answers

Answer:

answer for the question is 87

Plzzzzz Help me 10 points and will mark brainiest

Answers

Answer:

x=21

Step-by-step explanation:

We can use a ratio to solve

45          27

------ = ----------

35            x

Using cross products

45x = 35*27

45x = 945

45x/45 = 945/45

x  =21

Answer:

21

Step-by-step explanation:

The ratio of A to B is 45 / 35, which is equal to 9 / 7. To get from B to A, the ratio is 7 / 9.

(7 / 9) x 27

7 x 27 = 189, 189 / 9 = 21

Brainliest, please!

X/4- 2 =x/8+4

What value of x makes the equation in the reference image true ?

Answers

Answer:

The answer to your question is x=48

Step-by-step explanation: If you have any questions regarding my answer, tell me in the comments and I will answer them. Have a good day.

x/4−2=x/8+4

1/4x+−2=1/8x+4

1/4x−2=1/8x+4

1/4x−2−1/8x=1/8x+4−1/8x

1/8x−2=4

1/8x−2+2=4+2

1/8x=6

8*(1/8x)=(8)*(6)

x=48

PLS HELP MEEE I NEED HELP TO PASS PYTHAGOREAN THEOREM

Answers

Answer:

b=12

Step-by-step explanation:

b^2+35^2=37^2

b^2+1225=1369

b^2=144

b=12,-12

Lengths can't be negative, so b=12

Answer:

b=12

Step-by-step explanation:

The Pythagorean theorem is

a^2+b^2 = c^2  where a and b are the legs and c is the hypotenuse

35^2 + b^2 = 37^2

1225+b^2 =1369

b^2 =1369-1225

b^2 = 144

Taking the square root of each side

sqrt(b^2) = sqrt(144)

b=12

Simplify.
2(x + 2) -x+3(-x-2) - (2x + 4)

Answers

Answer:

-4x - 6

Step-by-step explanation:

2(x + 2) - x + 3(-x-2) - (2x + 4)

=> 2x + 4 - x + -3x - 6 - 2x - 4

=> 2x - 2x - x - 3x + 4 - 6 - 4

=> -4x - 6

Can someone please help me with this problem!!

Graph g(x)= -5^x +5

Answers

If you’re asking me to graph that, here it is!
If you are trying to find the X and Y intercept the answer is

X-intercept-(1,0)
Y -intercept-(0,4)
I hope this helped ! :)

The sum of two numbers is 75. If one exceeds the other by 35, find the numbers.

Answers

Answer:

20 and 55

Step-by-step explanation:

Let's assume, we have two numbers 'x' and 'y'.

Now, all we know is:

one number exceeds the other by 35 (i.e  their difference is 35)sum of two numbers is 75

So we form two equations:

x + y = 75_____(i)

and x - y =  35

so, x = y+35 _____(ii)

Put the value of x from (ii) into (i)

x + y = 75

y + 35 + y = 75

2y = 40

y = 20

Put value of y back in (ii)

x = y+35

x = 20 + 35

x = 55

The first number is 20, and the second number is 55.

Given that the sum of two numbers is 75, one exceeds the other by 35, we need to find the numbers.

Let's assume the first number is x.

According to the problem, the second number exceeds the first by 35, so the second number can be expressed as (x + 35).

The sum of the two numbers is given as 75, so we can set up the equation:

x + (x + 35) = 75

Simplifying the equation, we get:

2x + 35 = 75

Subtracting 35 from both sides:

2x = 40

Dividing both sides by 2:

x = 20

Therefore, the first number is 20, and the second number is (20 + 35) = 55.

Learn more about numbers click;

https://brainly.com/question/24908711

#SPJ4

Write an inequality and show on a number line all numbers:

greater than (-3) but less than or equal to 3

(enter the inequality in terms of x)

Eg. x<-3; 3 7

Answers

Answer:

The inequality is:

- 3 < x ≤ 3

And the graph is attached

Greater than -3

x>-3-3<x

Less than or equal to 3

x≤3

For number line

put a open dot at -3

put a close dot at 3

join them

factorise the quadratic equation x square + 2 x + 36​

Answers

Answer:

Click on the link below to view the answer with the explanation in futher detail

Explanation:

Click the attached link to view the answer

13. Find a four digit number that is divisible by 3,
5, and 8.

Answers

Answer:

3×5×8=120

Step-by-step explanation:

divisibility and prime number

heeeellllp...

...

...

...

...

...

...

Answers

Answer:

didn't shade all of them

Other Questions
What is the area of this figure A small ball of uniform density equal to 1/2 the density of water is dropped into a pool from a height of 5m above the surface. Calculate the maximum depth the ball reaches before it is returned due to its bouyancy. (Omit the air and water drag forces). which situation would most likely cause demand for apples to increase Define external digestion Which of the following is most likely the next step in the series? A sample of brass, which has a specific heat capacity of , is put into a calorimeter (see sketch at right) that contains of water. The temperature of the water starts off at . When the temperature of the water stops changing it's . The pressure remains constant at . Calculate the initial temperature of the brass sample. Be sure your answer is rounded to significant digits. A 1.25 kg block is attached to a spring with spring constant 17.0 N/m . While the block is sitting at rest, a student hits it with a hammer and almost instantaneously gives it a speed of 49.0 cm/s . What are You may want to review (Pages 400 - 401) . Part A The amplitude of the subsequent oscillations What did you expect in Statistic Methods in Nursing Research? Based on a sample survey, a company claims that 86% of their customers are satisfied with their products. Out of 1,100 customers, how many would you predict to be satisfied? Which of the following shows the coordinates of A (6, 12)after reflection over the y-axis? What would be the best topic sentence for the following paragraph? The Mac has a much design and can run multiple programs faster. This makes the Mac the ideal choice if you want to edit music or videos. You are also less likely to receive a virus with a Mac but do Apple provides fantastic customer supportNow the cheaper but you get what you pay for Put out the extra money and the superior computer Help me find the mistake in this sentence What are Social rites and culture regarded as?! Which of the following events started the Red Scare?the Bay of Pigs Invasionthe Vietnam Wara Soviet defector turned over evidence that the Soviet Union was trying to infiltrate the U.S. governmentthe Korean War Please help me ASAP! and thank you 5. Biking at a constant rate, Elise travels35 miles in 7 hours. If she travels at thesame speed, how far will Elise bike in9 hours?A 5 miB 44 miC 45 miD 63 mi A position statement does two things: It names the issue you are writing about and _____. Graph 3 points on the line y = x+6Y=x+6 X+y=8 what percent of 98 million is 7740 Plz anyone help me I need help.. This is Bangladesh language who can I will give them 13 point but who doesn't no if give wrong answer for point I will report them who can give answer I will give them brilliantiest